SlideShare a Scribd company logo
2007 ACR Diagnostic Radiology In-Training Exam Rationales
Section IX Musculoskeletal Radiology
179. You are shown a lateral radiograph of the spine and AP radiograph of the
pelvis of a 60 year old man (Figures 1A and 1B). What is the MOST LIKELY
diagnosis?
A. Diffuse idiopathic skeletal hyperostosis
B. Ankylosing spondylitis
C. Psoriatic arthritis
D. Ossification of the posterior longitudinal ligament
RATIONALES:
A. Correct. There is ossification of the anterior longitudinal ligament involving
numerous (>4) levels with relative preservation of the disc spaces and there is
extensive bone proliferation at the pelvis, at the trochanters, ischial tuberosities,
iliac spines and iliac crests without sacroiliitis. These are typical features of DISH.
The most common site of involvement is the vertebral column, Forestier’s
disease, which characteristically involves the thoracic region. The preservation of
the disc spaces and extensive spinal involvement help distinguish DISH from
degenerative disc disease. Normal S-I joints exclude a diagnosis of AS which
may otherwise demonstrate a similar proliferative enthesopathy at the pelvis.
B. Incorrect. Sacroiliitis is a diagnostic criterion for ankylosing spondylitis and the
S-I joints in this case are normal. The syndesmophytes of AS represent
ossification of the annulus fibrosis of the disc and, therefore, are thinner and
more vertical than the ossification of the anterior longitudinal ligament seen in
DISH.
C. Incorrect. The paravertebral ossification of Psoriatic arthritis, like that of
Reiter’s syndrome, is relatively larger, bulky or irregular, unilateral or asymmetric
in distribution and to the side of the vertebral column. The S-I joints may or may
not be involved. Proliferative ensethopathy at the pelvis may be present.
D. Incorrect. The posterior longitudinal ligament is not affected in this patient. Although
ossification of the posterior longitudinal ligament (OPLL) may be seen at the thoracic
spine, with or without cervical spine involvement, isolated cervical spine involvement is
more typical. There are no associated abnormalities at the pelvis.
References:
Miller and Schweitzer. Diagnostic Musculoskeletal Imaging. McGraw-Hill, NY,
2005.
Resnick. Diagnosis of Bone and Joint Disorders. 4th
ed. W.B. Saunders, NY,
2002
2007 ACR Diagnostic Radiology In-Training Exam Rationales
180. You are shown an axial fat-suppressed fast spin echo T2-weighted image
of a 16-year-old boy following knee trauma (Figure 2). What is the MOST
LIKELY diagnosis?
A. Ruptured anterior cruciate ligament
B. Hyperextension injury
C. Patellofemoral dislocation
D. Direct impaction (“dashboard”) injury
RATIONALES:
A. Incorrect. Rupture of the ACL, caused by a valgus rotational injury, is
associated with bone contusions at the lateral aspect of the knee, consisting of
the weight-bearing portion of the lateral femoral condyle and the posterior aspect
of the lateral tibial plateau.
B. Incorrect. Hyperextension of the knee causes “kissing” bone bruises of the
anterior aspect of the femoral condyles and tibial plateau.
C. Correct. There is impaction fracture at the medial aspect of the patella, bone
bruise at the lateral aspect of the lateral femoral condyle and partial tear of the
medial patella retinaculum. As the patella dislocates laterally, the medial
retinaculum is stretched. As the patella reduces spontaneously, the medial
aspect of the patella impacts the lateral aspect of the lateral femoral condyle.
This is often not suspected clinically.
D. Incorrect. A direct blow to the anterior aspect of the knee may cause posterior
dislocation of the femur or tibia.
References:
Sanders TG, Medynski MA, Feller JF, Lawhorn KW. Bone Contusion Patterns of the
Knee at MR Imaging: Footprint of the Mechanism of Injury. Radiographics
2000;20:S135-S151
2007 ACR Diagnostic Radiology In-Training Exam Rationales
181. You are shown an AP radiograph of a 30-year-old woman (Figure 3). What
structure is MOST LIKELY injured?
A. Posterior cruciate ligament
B. Iliotibial band
C. Medial meniscus
D. Anterior cruciate ligament
RATIONALES:
A. Incorrect. Posterior cruciate ligament tear is not a feature of this injury
pattern.
B. Incorrect. The iliotibial band inserts at Gerdy’s tubercle at the anterolateral
aspect of the proximal tibia, distal and more anterior to this avulsed attachment of
the lateral capsule.
C. Incorrect. Both the medial and lateral menisci may be torn as part of the
spectrum of injury, but not as often as the anterior cruciate ligament.
D. Correct. The lateral bony fragment is a “Segond” fracture and is due to avulsion of
the attachment of the lateral joint capsule resulting from a twisting injury associated with
varus stress. It is almost always associated with rupture of the anterior cruciate
ligament.
References:
Goldman AB, Pavlov H, Rubenstein D. The segond fracture of the proximal tibia:
A small avulsion that reflects major ligamentous damage. AJR 1988; 151:1163-
1167
2007 ACR Diagnostic Radiology In-Training Exam Rationales
182. You are shown an oblique coronal T2-weighted gradient echo image of a
50-year-old man with shoulder pain (Figure 4). Which one of the following is the
cause of the abnormality?
A. Calcium hydroxyapatite
B. Calcium pyrophosphate dihydrate
C. Monosodium urate
D. Hemosiderin
RATIONALES:
A. Correct. There is focal signal void at the suprapinatus tendon The capsular,
tendinous, ligamentous and bursal tissues about the shoulder are the most
common sites of peri-articular calcific deposits. The supraspinatus tendon is the
most frequent site of calcification, usually at the insertion near the greater
tuberosity. Although other crystals may deposit here, most are calcium
hydroxyapatite.
B. Incorrect. CPPD deposition may be present in synovium, cartilage, capsule,
tendon, ligament and bursae. Involvement of the supraspinatous tendon is not
uncommon and there may be associated HAA deposition. Deposition in cartilage,
however, is more typical.
C. Incorrect. Monosodium urate deposition may occur in the peri-articular soft
tissues including the joint capsule, tendon, ligament and bursa but is much more
common in the synovium and articular cartilage. There is a predilection for the
lower extremity.
D. Incorrect. Hemosiderin deposits within the synovium not the tendons and may be
secondary to resolving hematoma or recurrent hemarthrosis as in pigmented
villonodular synovitis or hemophilia. It is noted with inflammatory arthritis as well.
References:
Manaster, Disler, May. Musculoskeletal Imaging. The Requisites. 2nd
ed. Mosby,
St. Louis MO, 2002.
Resnick, Niwayama. Diagnosis of Bone and Joint Disorders. W.B. Saunders.
2002,Philadelphia, PA. Fourth Ed.
2007 ACR Diagnostic Radiology In-Training Exam Rationales
183. You are shown an AP radiograph of the left hip of a 75-year-old man with
hip pain (Figure 5). What is the MOST LIKELY diagnosis?
A. Giant cell tumor
B. Transient osteoporosis of the hip
C. Plasmacytoma
D. Paget’s Disease
RATIONALES:
A. Incorrect. Giant cell tumor is a sub-articular, focal, lytic lesion, often eccentric,
and demonstrating expansile remodeling. Patients are usually 20-45 years of
age. Extension of the neoplasm into the shaft without associated deformity of the
bone would be unusual.
B. Incorrect. Transient osteoporosis is a self-limited disorder, associated with
transient bone marrow edema. It may be the sequela of subchondral insufficiency
fracture. The radiographic appearance is that of focal osteoporosis at the
femoral head with less extensive involvement of the femoral neck and
acetabulum without involvement more distally and without the advancing edge
seen in the test case.
C. Incorrect. Plasmacytoma is a common lytic lesion of the skeleton in older patients.
The lesions however are more focal with bone destruction appearing as a geographic
lytic area sometimes with expansile remodeling, none of which is seen in the test case.
Diffuse osteopenia is another manifestation.
D. Correct. The lytic phase of Paget’s disease, typically begins at the end of a bone, or
an apophysis, and advances towards the other end with a well-defined advancing edge,
blade-like, V or wedge or flame shaped in appearance. The lytic nature of the process
is secondary to bone resorption, not destruction or replacement, and therefore contour
abnormalities are not prominent as in the test case where the only deformity is
secondary to fracture.
References:
Smith,Murphey et al. Radiologic Spectrum of Paget Disease of Bone and Its
Complications with Pathologic Correlation. AFIP Archives. Radiogaphics 2002.22:1191-
1216.
Greenspan. Orthopaedic Radiology. Lipincott Williams Wilkins. Philadelphia, PA. Third
Ed.
Resnick, Niwayama. Diagnosis of Bone and Joint Diaorders. W.B. Saunders.
Philadelphia, PA. Fourth Ed, 2002.
2007 ACR Diagnostic Radiology In-Training Exam Rationales
184. You are shown coronal (Figure 6A) and axial (Figure 6B) fat-suppressed T1-
weighted MR arthrographic images of a 24-year-old man with right shoulder pain.
Which of the following is the MOST LIKELY diagnosis?
A. Hill-Sachs lesion
B. Bankart lesion
C. Perthes lesion
D. SLAP lesion
RATIONALES:
A. Incorrect. The Hill-Sachs fracture deformity is a fracture at the posterior lateral
aspect of the humeral head, superiorly, acquired following anterior shoulder
dislocation and subsequent impaction at the anterior inferior glenoid. It is best
seen on axial images at the level of the coracoid or higher. It should not be
confused with the normal posterior humeral groove, seen more distally.
B. Incorrect. A Bankart lesion is an anterior labral avulsion at the inferior
glenohumeral ligament attachment with rupture of the anterior scapular
periosteum and marked periosteal stripping. It is associated with anterior
shoulder dislocation. Although contrast is seen beneath the detached labrum, it is
confined by the intact overlying periosteum.
C. Correct. A Perthes lesion is a Bankart variant, an anterior labral avulsion at
the inferior glenohumeral ligament attachment with intact scapular periosteum
and mild periosteal stripping. The intra-articular contrast is seen beneath the
labrum at the detachment site and extends medially beneath the mildly stripped,
but otherwise intact, periosteum.
D. Incorrect. The superior labrum is intact.
References:
Stoller DW. Shoulder. In: Stoler DW, Tirman PF, Bredella MA, eds. Orthopaedics. Salt
Lake City, UT: Amirsys, 2004: 58-93.
2007 ACR Diagnostic Radiology In-Training Exam Rationales
185. A standard spin echo pulse sequence with TR = 4000 ms and TE = 90 ms
will have image contrast chiefly dominated by ____________ weighting.
A. T1
B. Proton density
C. T2 X
D. T2*
RATIONALES:
A. Incorrect: Standard spin-echo sequences rely on the fact that tissues with
short (long) T1 will typically have a short (long) T2. T1 is the spin-lattice
relaxation constant, which describes the time required for re-establishment of
63% of the longitudinal magnetization, and T2 is the spin-spin relaxation
constant, which describes the time required for decay of the transverse
magnetization to 37% of its original peak amplitude. T1 contrast is manifested by
selecting a TR time that maximizes differences in the T1 characteristics of the
tissues, and is typically between about 300-700 ms for a standard spin-echo
sequence. In order to reduce the effects of T2 decay, a short TE (<10 ms) is
required. The stem indicates values much longer than would generate T1
contrast.
B. Incorrect: TR is considered “long” in a standard spin-echo pulse sequence
above about 800 to 1000 ms, where the longitudinal magnetization differences
are manifested chiefly as spin-density (proton-density) variations, with minimal
T1 weighting. While a TR of 4000 ms can certainly result in spin-density
weighting, the other part of signal generation is the spin-spin decay of transverse
magnetization, which requires a very short TE (<10 ms). The TE of 90 ms is
considered to be long, giving rise to differences in the T2 characteristics of the
tissues.
C. Correct: TR is considered “long” in a standard spin-echo pulse sequence
greater than 800 to 1000 ms, where the longitudinal magnetization differences
are manifested chiefly as spin-density (proton-density) variations, to reduce any
T1 weighting effects. For TR = 4000 ms, there is little or no T1 weighting.
Transverse magnetization losses (spin-spin decay) are due to T2 effects; by
allowing the decay to occur over a relatively long time prior to producing an echo,
more T2 contrast will result. For spin-echo sequences, TE > 50 ms is considered
long, and will permit more transverse decay to occur, resulting in the
manifestation of T2 contrast.
D. Incorrect: T2* weighting is not apparent with a standard spin-echo pulse sequence
because of the 180° refocusing pulse, which causes the de-phasing spins to be subject
to external magnetic inhomogeneities in the opposite direction, which cancels the de-
phasing effect in the reformed echoes.
2007 ACR Diagnostic Radiology In-Training Exam Rationales
References: Bushberg JT, Seibert JA, Leidholdt EM, Boone JM. The Essential Physics
of Medical Imaging, 2nd
Edition, Chapter 14, p. 399.
186. Concerning sacroiliitis, which of the following is MOST LIKELY to be mono-
articular?
A. Tuberculosis
B. Progressive systemic sclerosis
C. Inflammatory bowel disease
D. Reiter’s syndrome
RATIONALES:
A. Correct. Tuberculous spondylitis is the most common form of skeletal
tuberculosis. Extra-spinal articular disease typically involves large joints such as
the knee and hip but any joint may be involved. Monoarticular involvement is
characteristic. Osteomyelitis without septic arthritis is relatively uncommon.
B. Incorrect. Sacroiliitis is not a feature of progressive systemic sclerosis or
scleroderma.
C. Incorrect. The sacroiliitis associated with ulcerative colitis and Crohn’s
disease is identical to that of ankylosing spondylitis, bilateral and symmetric.
D. Incorrect. Sacro-iliitis in Reiter’s syndrome is common. The involvement tends to be
bilateral, symmetric or asymmetric. Asymmetric and rarely,unilateral involvement is
more common in Reiter’s syndrome and psoriatic arthritis than ankylosing spondylitis.
References:
Miller and Schweitzer. Diagnostic Musculoskeletal Imaging. McGraw-Hill, NY,
2005.
Chew. Skeletal Radiology: The Bare Bones. 2nd
ed. Williams and Wilkins,
Baltimore, 1997.
Resnick. Diagnosis of Bone and Joint Disorders.4th
ed. W.B. Saunders, NY, 2002
2007 ACR Diagnostic Radiology In-Training Exam Rationales
187. Concerning chronic renal disease, which of the following is MOST closely
associated?
A. Neuropathic osteoarthropathy
B. Tarsal tunnel syndrome
C. Destructive spondyloarthropathy
D. Hypertrophic osteoarthropathy
RATIONALES:
A. Incorrect. Neuropathic osteoarthropathy may result from several underlying
disorders including diabetes mellitus, syphilis, and numerous congenital
insensitivity to pain syndromes. Radiographic findings of bone destruction and
fragmentation may mimic the destructive spondyloarthropathy associated with
renal dialysis.
B. Incorrect. Carpal tunnel syndrome is a known complication of patients
undergoing hemodialysis. This has been attributed to edema, venous congestion
and subsequent compression of the median nerve related to the access site.
Amyloid deposition, B2-microglobulin, in the synovium also results in such
compression. Tarsal tunnel syndrome is not a clinical manifestation related to
chronic renal disease.
C. Correct. Patients undergoing renal dialysis for at least 2-3 years may develop
a destructive spondyloarthropathy, most common at the cervical region. This may
or may not be symptomatic. Radiographic findings of disc space loss, endplate
erosion and reactive sclerosis mimic septic spondylitis, neuropathic
spondyloarthropathy and CPPD deposition disease. Although chronic
hyperparathyroidism and ligamentous laxity may be contributing factors, amyloid
deposition seems primary.
D. Incorrect. Hypertrophic osteoarthropathy, usually associated with pulmonary
disorders (hypertrophic pulmonary osteoarthropathy, HPO) most notably bronchogenic
carcinoma, is a triad of painful swollen joints, clubbing and periosteal new bone
formation. Although this is associated with numerous disorders including inflammatory
bowel disease, biliary cirrhosis, chronic lung diseases, and congenital cyanotic heart
disease, there is no association with chronic renal disease.
References: Miller and Schweitzer. Diagnostic Musculoskeletal Imaging.
McGraw-Hill, NY, 2005.
Manaster, Disler, May. Musculoskeletal Imaging. The Requisites. 2nd
ed. Mosby,
St. Louis, 2002.
Resnick. Diagnosis of Bone and Joint Disorders.4th
ed. W.B. Saunders, NY, 2002
2007 ACR Diagnostic Radiology In-Training Exam Rationales
188. Concerning mixed connective tissue disease, which of the following is
TRUE?
A. Males and females are equally affected.
B. It is a combination of scleroderma, systemic lupus erythematosus, and
polyarteritis nodosa.
C. Erosive disease is not characteristic.
D. Serology is essential to diagnosis.
RATIONALES:
A. Incorrect. Approximately 80% of patients are woman.
B. Incorrect. Mixed connective tissue disease (MCTD) is a disorder
characterized by clinical abnormalities typical of SLE (systemic lupus
erythematosus), PSS (progressive systemic sclerosis or scleroderma),
dermatomyositis and rheumatoid arthritis.
C. Incorrect. Joint involvement is typical resembling the changes of rheumatoid
arthritis. Occasionally, nonerosive deformities similar to SLE are seen.
D. Correct. The one feature of MCTD which distinguishes it as a unique disorder is a
positive serologic test for antibody to the ribonucleoprotein (RNP) of extractable nuclear
antigen (ENA).
References: Miller and Schweitzer. Diagnostic Musculoskeletal Imaging.
McGraw-Hill, NY, 2005.
Greenspan, A. Orthopedic Radiology: A Practical Approach. Lippincott Williams
Wilkins, 3rd
. ed., 2000.
2007 ACR Diagnostic Radiology In-Training Exam Rationales
189. Concerning Blount’s disease, which of the following is TRUE?
A. It is also known as tibia valga.
B. Radiographic abnormalities are evident in the first 2 years of life.
C. The disorder can occur in infants, children and adolescents.
D. It is usually self-limited and requires no treatment.
RATIONALES:
A. Incorrect. Blount’s disease is also known as tibia vara. The disorder affects
the medial aspect of the proximal tibia. The deformity consists of varus
angulation and internal rotation at the proximal tibial metaphysis. Factors
contributing to the disorder include varus stress, focal growth suppression and
disruption of endochondral ossification.
B. Incorrect. Radiographic abnormalities in the infantile form rarely are evident
before 2 years of age at which time they resemble physiologic bowing. The varus
deformity, however, occurs at the metaphysis, not the knee. Eventually, the
medial metaphysis becomes depressed and an osseous excrescence or
outgrowth may develop.
C. Correct. The early onset or infantile group occurs in children less than three
years of age. The late onset group consists of a juvenile and an adolescent form.
Deformity is more likely in the younger age groups.
D. Incorrect. The natural history of the disease is that of irreversible deformity
resulting from changes at the growth plate. In the infantile form, orthotic
bracing is usually the first line of treatment. If this fails, an osteotomy is required.
In the adolescent form, treatment is usually surgical.
References:
Bradway JK, Klassen RA, Peterson HA. Blount disease: a review of the English
literature. J Pediatr Orthop 1987; 7(4):472-480
Blounts Disease. Wheeless' Textbook of Orthopaedics. Duke Orthopaedics.
www.wheelessonline.com/ortho/blounts_disease
Resnick, Niwayama. Diagnosis of Bone and Joint Disorders. W.B. Saunders.
2002,Philadelphia, PA. Fourth Ed
2007 ACR Diagnostic Radiology In-Training Exam Rationales
190. Concerning aneurysmal bone cysts, which of the following are associated?
A. Fibrous dysplasia
B. Osteoid osteoma
C. Enchondroma
D. Plasmacytoma
RATIONALES:
A. Correct. Aneurysmal bone cysts are often superimposed upon existing
benign and malignant tumors most notably giant cell tumor, nonossifying fibroma,
fibrous dysplasia, chondroblastoma, osteoblastoma, osteosarcoma,
chondrosarcoma and hemangioendothelioma. In the case of telangiectatic
osteosarcoma, the two different components of the lesion may appear similar or
identical with imaging. This may result in errors with percutaneous biopsy if only
portions of the cyst are sampled. Nonossifying fibroma is a benign, cortically
based lesion that is usually found in long tubular bones. The etiology is argued,
but many feel the nonossifying fibroma to be the sequelae of unrecognized
trauma to the periosteum that leads to focal hemorrhage. Aneurysmal bone
cysts are also felt by many to represent prior trauma, not neoplasia. It is
therefore intuitive that the two lesions could be found in the same location.
Fibrous dysplasia is a hamartomatous disorder of osteoblasts that fail to undergo
differentiation. The monostotic form is most common. Skin and endocrine
anomalies may be associated with the polyostotic form. Although fibrous
dysplasia may rarely be associated with malignant transformation, new
symptoms or growth of a known focus of fibrous dysplasia often signifies the
development of a superimposed aneurysmal bone cyst.
B. Incorrect. Osteoid osteoma is a benign bone forming neoplasm usually
cortically based and extra-articular, with characteristic reactive sclerosis.
Intracapsular, intramedullary and subperiosteal lesions also occur. Osteoid
osteomas that are intra-articular incite little or no sclerosis. Intra-medullary and
subperiosteal lesions provoke variable degrees of sclerosis. There is no
association with ABC. Osteoblastoma, however, is associated.
C. Incorrect. Enchondroma is a benign cartilage forming neoplasm within the
medullary canal. Such lesions on the surface of the bone are known as
superiosteal or juxta-cortical chondromas. They are most common at the fingers.
There is no association with ABC.
D. Incorrect. Plasmactyoma represents the solitary form of myeloma. While laboratory
teats may be negative at the time of diagnosis, many patients will eventually develop
multiple myeloma. The lesions are usually lytic, and may be expansile or non-
expansile. Rarely, a plasmacytoma may be blastic. An association between
plasmacytoma and aneurysmal bone cyst has not been described.
2007 ACR Diagnostic Radiology In-Training Exam Rationales
References:
Mirra JM, Picci P, Gold RH. Bone Tumors: Clinical, Radiologic, and Pathologic
Correlations. Lea & Febiger, Philadelphia, PA, 1989
191. Which of the following tumors is characterized by a subarticular location?
A. Osteoblastoma
B. Clear cell chondrosarcoma
C. Chondromyxoid fibroma
D. Aneurysmal bone cyst
RATIONALES:
A. Incorrect. The most common location for osteoblastoma is in the dorsal elements of
the spine where forty percent are found. The remaining are usually located in the long
bones and phalanges. An epiphyseal or subarticular location is rare. Osteoblastoma is
usually expansile, and may be associated with neurologic deficit.
B. Correct. Few lesions have a propensity for the end of the bone, epiphysis or
apophysis (epiphysis equivalent). Giant cell tumor of bone and chondroblastoma
are oth such lesions. Approximately 5% of giant cell tumors are malignant and
associated with lung metastases. Otherwise, both of these are considered
benign. The malignant tumor that is found at the end of the bone, subarticular, is
clear cell chondrosarcoma. This is a slow growing tumor, and symptoms may be
present for over five years before diagnosis. The tumor may be lobulated and
calcified, or lytic and ill defined.
C. Incorrect. As the name implies, chondromyxoid fibroma is a benign tumor
composed of chondroid, myxoid and fibrous tissue. Three-fourths are found in
the lower extremities. Approximately half involve long bones, and ninety-five
percent of those are found in the metaphysis. The remaining five percent are
diaphyseal.
D. Incorrect. Although aneurysmal bone cysts may be engrafted on epiphyseal
or subarticular lesions such as giant cell tumor and chondroblastoma, the most
common locations are the metaphysis of long bones and the dorsal elements of
the vertebrae. Approximately one third of aneurysmal bone cysts are secondary,
associated with other entities. The remaining may represent primary aneurysmal
bone cysts, or may reflect complete destruction of the entire pre-existing tumor
by the aneurysmal bone cyst.
References:
2007 ACR Diagnostic Radiology In-Training Exam Rationales
Mirra JM, Picci P, Gold RH. Bone Tumors: Clinical, Radiologic, and Pathologic
Correlations. Lea & Febiger, Philadelphia, PA, 1989
192. Concerning osteosarcoma, what type has the worst prognosis?
A. Secondary
B. Telangiectatic
C. Periosteal
D. Parosteal
RATIONALES:
A. Correct. Secondary osteosarcoma may be seen following radiation therapy,
or may be associated with underlying bone lesions such as Paget's disease,
fibrous dysplasia, bone infarction and chronic osteomyelitis. Other types of
sarcoma may also be seen with these conditions, including fibrosarcoma and
malignant fibrous histiocytoma. The prognosis for these secondary sarcomas is
often grave.
B. Incorrect. Telangiectatic osteosarcoma is an entirely lytic lesion, reflecting
minimal osteoid production in the tumor. Far less common than conventional
osteosarcoma, it was previously felt that the telangiectatic form carried a worse
prognosis. More recent studies have shown a similar or even better prognosis,
as the telangiectatic form is especially sensitive to preoperative chemotherapy.
C. Incorrect. Periosteal osteosarcoma is a chondroblastic form of osteosarcoma
that arises either in the periosteum or in the outer cortex, and is usually
diaphyseal in location. The lesions show a sunburst pattern of mineralization.
They are usually considered to be intermediate grade, between high grade
central and low grade parosteal osteosarcomas.
D. Incorrect. The parosteal (juxtacortical) osteosarcoma is found on the outer surface
of the cortex. These tumors are well differentiated, with a prognosis better than other
forms of osteosarcoma. A long-tern survival of eighty to ninety percent is seen if the
tumor has not de-differentiated. Two thirds of parosteal osteosarcomas are located
along the distal femoral shaft. Most are heavily ossified.
References:
Unni KK, ed. Dahlin's Bone Tumors: General Aspects and Data on 11, 087
cases, 5th ed. Lippincott-Raven Publishers, Philadelphia, PA, 1996
2007 ACR Diagnostic Radiology In-Training Exam Rationales
193. Concerning osteopoikilosis, which of the following is TRUE?
A. Patients often complain of restricted range of motion at the joints involved.
B. Histologically, the lesions are easily confused with metastatic prostate cancer.
C. The patient’s serum alkaline phosphatase may be mildly elevated.
D. Transmission is autosomal dominant.
RATIONALES:
A. Incorrect. Osteopoikilosis is characterized by multiple bone islands or
enostoses with a periarticular distribution. This is an asymptomatic, incidental
condition not to be confused with other sclerotic lesions most notably blastic
metastases. It may be considered a type of sclerosing dysplasia similar in this
regard to osteopathia striata (Voorhoeve’s disease).
B. Incorrect. Histologically, the lesions are benign bone islands. They consist of
compact lamellar bone with haversian systems. The bone is uniform with regular
cement lines and no evidence of cartilage. The characteristic radiographic
appearance includes numerous, small, round or ovoid (long axis parallel to long
tubular bone) sclerotic lesions, with spiculated margins and periarticular,
symmetric distribution favoring long tubular bones, the carpus, tarsus, pelvis and
scapulae. Lesions may increase or decrease in size and number although this
phenomenon is more common in children. Bone scans are usually normal.
C. Incorrect. Laboratory examinations are normal.
D. Correct. Both inherited and sporadic cases of osteopoikilosis have been
described. There is an autosomal dominant pattern of transmission and
penetrance is high.
References:
Lagier R, Mbakop A, Bigler A. Osteopoikilosis: a radiological and pathological
study. Skeletal Radiol 1984; 11: 161-168
Resnick, Niwayama. Diagnosis of Bone and Joint Disorders. W.B.
Saunders. 2002, Philadelphia, PA. Fourth Ed
2007 ACR Diagnostic Radiology In-Training Exam Rationales
194. Which of the following synovial spaces normally communicate with each
other?
A. The glenohumeral joint and the subacromial bursa
B. The pisiform recess and the radiocarpal joint
C. The ankle joint and the peroneal tendon sheath
D. The posterior subtalar joint and the Achilles tendon sheath
RATIONALES:
A. Incorrect. Communication between the glenohumeral joint and the
subacromial bursa (more accurately, the subacromial/subdeltoid bursal complex)
indicates the presence of a full thickness rotator cuff tear. This principal is the
basis for diagnosis of such tears on both conventional and MR glenohumeral
arthrography.
B. Correct. Either space may be entered for wrist arthrography. Excessive fluid
in the pisiform recess should not be considered abnormal if there is a radiocarpal
joint effusion.
C. Incorrect. The ankle joint may normally communicate with the tendon sheaths
of the flexor digitorum longus and flexor hallucis longus at the medial aspect of
the ankle. Communication with the lateral tendon sheath (common peroneal)
implies a tear of the calcaneofibular ligament.
D. Incorrect. The Achilles tendon does not have a true tendon sheath. Instead, it
is surrounded by connective tissue called a paratenon. This provides for the
normal gliding action of the tendon.
References:
Arndt RD, Horns JW, Gold RH. Clinical Arthrography. 2nd
ed. Williams & Wilkins,
Baltimore, MD. 1985.
2007 ACR Diagnostic Radiology In-Training Exam Rationales
195. Concerning pars interarticularis defects, which of the following is TRUE?
A. Symptomatic spondylolisthesis is typically associated.
B. Associated spondylolisthesis is most common at L5-S1.
C. They represent an acute injury.
D. Magnetic resonance imaging is the preferred imaging method.
RATIONALES:
A. Incorrect. Only approximately twenty-five percent of patients with
spondylolysis develop spondylolisthesis. Progression to significant slippage is
uncommon.
B. Correct. Spondylolisthesis associated with spondylolysis is most common at
L5-S1, approximately 90% of cases. The remainder usually occur at L4-L5.
Spondylolisthesis associated with degenerative disease is most common at L4-
L5.
C. Incorrect. Pars interarticularis defects are considered to represent fatigue
stress fractures. Most are believed to develop in childhood. They are especially
common in athletic children.
D. Incorrect. Imaging should begin with conventional radiographs. The lateral
and oblique views are nearly equally sensitive in diagnosing the defect. CT also
easily displays the lesion. SPECT imaging can be useful in identifying pars
interarticularis defects that are symptomatic. Magnetic resonance imaging plays
little role. Difficulty in recognizing an intact pars interarticularis leads to a low
positive predictive value.
References:
Garry JP, McShane J. Lumbar spondylolysis in adolescent athletes. J Fam Pract.
1998; 47(2): 145-149
Standaert CJ, Herring SA. Spondylolysis: a critical review. Br J Sports Med
2000; 34(6): 415-422
Resnick. Diagnosis of Bone and Joint Disorders.4th
ed. W.B. Saunders, NY, 2002
2007 ACR Diagnostic Radiology In-Training Exam Rationales
196. Concerning the vertebral column, which of the following is TRUE?
A. Paget’s Disease spares the posterior elements.
B. Hematogenous infection preferentially seeds the vertebral body.
C. Soft tissue extension excludes the diagnosis of hemangioma.
D. The inflammatory spondyloarthropathies spare the intervertebral discs.
RATIONALES:
A. Incorrect. Paget‘s disease of the spine typically involves the vertebral body and the
posterior elements. This may be a helpful diagnostic clue when differentiating Paget ’s
disease, blastic metastases/ lymphoma and hemangioma.
B. Correct. Hematogenous infection of the spine begins as an osteomyelitis near
one of the vertebral body endplates. Typically, the infection then spreads across
the disc to the adjacent vertebral body resulting in destruction and erosion of the
endplates and disc. Less commonly, the infection may spare the disc tracking
beneath the anterior or posterior longitudinal ligaments, a pattern usually seen
with tuberculous involvement. Despite the use of the term “discitis,” infection
involving the spinal column rarely originates within the disc itself.
C. Incorrect. Large hemangiomata may extend beyond the cortex into the para
vertebral soft tissues and be symptomatic.
D. Incorrect. The inflammatory spondyloarthropathies comprise the sero-negative
disorders which typically involve synovial, cartilaginous and fibrous joints. The
cartilaginous joints in the adult skeleton are the intervertebral discs and the symphysis
pubis. Syndesmophyte formation and subsequent ossification of the intervertebral disc
in patients with ankylosing spondylitis is an example of such involvement.
References:
Resnick,Niwayama. Diagnosis of Bone and Joint Disorders. W B
Saunders,Philadelphia, PA . Fourth Ed. 2002
2007 ACR Diagnostic Radiology In-Training Exam Rationales
197. Concerning the osteochondroses, which of the following entities represents
normal development?
A. Kohler’s
B. Scheurermann’s
C. Kienbock’s
D. Sever’s
RATIONALES:
A. Incorrect. The osteochondroses compromise a group of entities characterized
by involvement of the epiphysis, apophysis or epiphyseoid bone with
radiographic findings of fragmentation, collapse and sclerosis suggesting
osteonecrosis. Although some of these entities represent osteonecrosis, others
are the sequela of abnormal stress and others represent normal development.
Kohler’s disease involves the tarsal navicular and is rare. It is a self-limited
disorder, difficult to distinguish from variations of normal ossification. When a
child is symptomatic and radiographic findings of flattening and sclerosis are
detected in a previously normal navicular bone, the diagnosis of osteonecrosis is
more certain.
B. Incorrect. Scheuermann’s disease represents a growth disturbance of the
spine characterized by multilevel anterior vertebral body wedging, vertebral body
endplate irregularities with Schmorl’s node formation and increasing thoracic
kyphosis. These changes are variable and most likely secondary to stress related
intraosseous displacement of disc material (cartilagenous node formation)
through cartilaginous endplates weakened on a congenital or traumatic basis.
C. Incorrect. Kienbock’s disease represents osteonecrosis of the lunate seen in
patients 20-40 years of age.
D. Correct. Sever’s phenomenon represents normal ossification of the calcaneal
apophysis.
References:
Resnick,Niwayama. Diagnosis of Bone and Joint Disorders. W B
Saunders,Philadelphia, PA . Fourth Ed. 2002
2007 ACR Diagnostic Radiology In-Training Exam Rationales
198. Concerning cervical spine fracture, which is characteristically associated
with acute, severe neurologic injury?
A. Jefferson
B. Extension teardrop
C. Hangman’s
D. Flexion teardrop
RATIONALES:
A. Incorrect. In a classic Jefferson fracture, the transverse ligament is intact
and no instability is present and because displacement of the bony fragments
occurs in a centripetal pattern during axial loading, cord damage is uncommon.
In one large series, no patient presented with neurologic symptoms.
B. Incorrect. This fracture is an avulsion fracture that arises from the
anteroinferior corner of the vertebral body caused by a hyperextension injury.
Retrolisthesis is often present but of minimal degree and only about 9% of
patients present with neurologic symptoms.
C. Incorrect. Bilateral fracture of the C2 pars interarticularis typically results from
hyperextension. Death from judicial hanging resulted from delayed
extension/distraction. Most cases today are secondary to motor vehicle accidents
with transient hyperextension and no distraction. There is usually anterior
subluxation of C2 on C3. Although unstable by nature, neurologic deficits are
uncommon. Without significant distraction, the cord is typically spared because
the acquired pars fracture allows for canal widening at a level where the cord
already has abundant room.
D. Correct. The flexion teardrop fracture is the most severe flexion injury
characterized by complete disruption of all ligaments, intervertebral disc and facet joints
at the level of injury and a large triangular fracture fragment consisting of the anterior,
inferior aspect of the involved vertebral body. There is neither ligamentous or skeletal
stability. This completely unstable fracture is associated with severe neurologic
symptoms in 87% of patients including complete quadriplegia, paraplegia, Brown-
Sequard syndrome, or anterior cord syndrome. It is caused by combined flexion and
axial loading and classically affects C5.
References:
Resnick, D. Physical Injury: Spine. In: Resnick D, ed. Diagnosis of Bone and Joint
Disorders. 4th
ed. Philadelphia, PA: W.B. Saunders, 2002: 2958-2978.
Yu JS. Hyperflexion teardrop fracture. In: Case Review – Musculoskeletal Imaging.
St. Louis, MO: Mosby, 2001:127-128.
2007 ACR Diagnostic Radiology In-Training Exam Rationales
199. Concerning injuries of the wrist, which is MOST severe?
A. Lunate dislocation
B. Scapholunate dissociation
C. Dorsal intercalated segment instability
D. Perilunate dislocation
RATIONALES:
A. Correct. The dislocation of carpal bones about the lunate, with or without
fracture, usually results from dramatic wrist hyperextension. High velocity trauma
or falls from substantial heights may produce forceful palmar tension and dorsal
compression that exceeds the limits of ligament and/or bony carpal constraints.
The progressive perilunar instability model for such injury predicts sequential
disruption of ligamentous attachments, forces transmitted from the radial to the
ulna aspect of the carpus. In stage I, there is disruption of the scapho-lunate
ligament and subsequent scapho-lunate dissociation. In stage II, the capitate and
scaphoid separate from the lunate and triquetrum . In stage III, there is
lunotriquetral dissociation allowing the entire carpus to separate from the lunate
almost always with dorsal displacement, hence dorsal perilunate dislocation. In
stage IV, the dorsal extrinsic ligaments fail and the lunate may dislocate volarly.
This represents the most severe form and highest degree of instability. Such
transmission of force purely about the lunate follows the so-called lesser arc
pathway. Perilunar forces may also be transmitted through adjacent bone, so-
called greater arc injuries. The scaphoid, trapezium, capitate, hamate and
triquetrum may fracture. As one might expect, there may be concomitant fracture
and ligamentous injury resulting in the perilunar dislocation stages described.
The lesser and greater arcs together comprise the vulnerable zone for all such
injuries.
B. Incorrect. Scapholunate dissociation represents a stage I lesser arc injury and
therefore is not considered severe. It is, however, a component of scapholunate
instability, the most frequent form of carpal instability. In its most general sense,
instability refers to a clinical perception by the patient that the joint can not
withstand normal loads. Scapholunate dissociation refers to the anatomic,
ligamentous disruption between scaphoid and lunate. This may only be
detectable with stress radiographs or fluoroscopy and hence is referred to as
dynamic instability. This may be readily apparent with routine radiographs, hence
static instability. The scapholunate angle may be normal as seen on lateral
radiographs. Only when the scaphoid assumes a flexed position however, is
there rotatory subluxation.
C. Incorrect. An intercalated segment is a middle segment of a three segment
system under compression. This middle segment is inherently unstable unless
stabilized by its connection to the other two segments, hence the proximal carpal
row. DISI (dorsal intercalated segment instability) refers to the dorsiflexed
posture or extension of all or part of the proximal carpal row, relative to the radius
2007 ACR Diagnostic Radiology In-Training Exam Rationales
and capitate. This may be associated with numerous injuries, with and without
dissociation of intrinsic or interosseous ligaments. It is less severe than lunate
dislocation.
D. Incorrect. Peri-lunate dislocation is a stage III perilunar injury and, therefore, less
severe than stage IV lunate dislocation.
References: Resnick, D. Physical Injury: Extraspinal Sites. In: Resnick D, ed.
Diagnosis of Bone and Joint Disorders. 4th
ed. Philadelphia, PA: W.B. Saunders, 2002:
2837-2842.
Viegas SF, Patterson RM, Ward K. Extrinsic wrist ligaments in the pathomechanics of
ulnar translation instability.J Hand Surg. 1995; 20A: 312-318.
Yu JS. Trans-scaphoid perilunate dislocation. In: Case Review – Musculoskeletal
Imaging. St. Louis, MO: Mosby, 2001:129-130.
Cooney,WP et at. The Wrist: Diagnosis and Operative Treatment. St. Louis, MO:
Mosby, 1998.
200. Portable radiographs taken with a film-screen system using a fixed
radiographic grid tend to have less contrast than the radiographs taken in the
radiography rooms. Which of the following factors is the MOST LIKELY cause
for the reduced contrast?
A. Use of lower kVp
B. Use of higher mAs
C. Use of Lower grid ratio
D. Use of higher speed film-screen system
RATIONALES:
A. Incorrect. Lower kVp would increase contrast.
B. Incorrect. Higher mAs would darken film, but not change contrast.
C. Correct. Lower grid ratio radiographic grid is used to minimize cutoff from
poor alignment, however the lower grid ratio yields less cleanup of the scatter
radiation.
D. Incorrect. Higher speed film-screen does not necessarily reduce contrast.
References:
A. Wolbarst, Physics of Radiology (1993), Chapter 20
J.T. Bushberg, et al., The Essential Physics of Medical Imaging (2002), Chapter
6
2007 ACR Diagnostic Radiology In-Training Exam Rationales
201. Which of the following structures is essential to maintaining the arch of the
foot?
A. Plantar fascia
B. Posterior tibial tendon
C. Spring ligament
D. Peroneal brevis tendon
RATIONALES:
A. Incorrect. The plantar fascia assists in supporting the midfoot arch but is
considered a secondary support. Rupture does not produce significant
separation of the plantar fascia and does not result in flatfoot deformity. The
clinical presentation is one of pain and swelling at the heel.
B. Correct. Complete rupture of the posterior tibial tendon frequently results in
pes planus deformity, the posterior tibial tendon being a primary support of the
midfoot arch. It is opposed by the peroneus brevis tendon which everts the heel
and abducts the foot. Posterior tibial tendon rupture allows the unopposed
peroneal brevis tendon to result in hindfoot valgus and forefoot abduction. Once
the calcaneus is in a valgus position, the Achilles tendon will further evert the
calcaneus. The talo-navicular joint is gradually disrupted as all the medial
ligaments become stretched and elongated.
C. Incorrect. The spring ligament extends from the sustentaculum tali to the
plantar aspect of the navicular. It supports the talar head and therefore the
longitudinal arch of the foot. It is thick and strong and almost never ruptures
during routine foot/ankle trauma. Although the spring ligament serves as a
secondary support of the midfoot arch, it is not a primary support and as long as
the posterior tibialis tendon is intact, the midfoot does not collapse.
D. Incorrect. As discussed, rupture of the peroneal brevis tendon would allow the
posterior tibial tendon to function unopposed . This injury is caused by repeated ankle
sprain or chronic peroneal subluxation. It does not lead to a pes planus deformity.
References: Yu JS. Posterior tibial tendon tear. In: Case Review – Musculoskeletal
Imaging. St. Louis, MO: Mosby, 2001:107-108.
Schweitzer ME, Karasick D. MR imaging of disorders of the posterior tibialis tendon.
AJR Am J Roentgenol. 2000; 175: 627-35.
Yu JS. Pathologic and post-operative conditions of the plantar fascia: review of MR
imaging appearances. Skeletal Radiol. 2000; 29: 491-501.

More Related Content

What's hot

23205011
2320501123205011
23205011
radgirl
 
23205056
2320505623205056
23205056
radgirl
 
23205029
2320502923205029
23205029
radgirl
 
23205062
2320506223205062
23205062
radgirl
 
23205008
2320500823205008
23205008
radgirl
 
23204964
2320496423204964
23204964
radgirl
 
23204977
2320497723204977
23204977
radgirl
 
23205065
2320506523205065
23205065
radgirl
 
23204961
2320496123204961
23204961
radgirl
 
23204955
2320495523204955
23204955
radgirl
 
23204943
2320494323204943
23204943
radgirl
 
23205001
2320500123205001
23205001
radgirl
 
23204952
2320495223204952
23204952
radgirl
 
Radiology most important signs sample questions based on neet pg , usmle, pla...
Radiology most important signs sample questions based on neet pg , usmle, pla...Radiology most important signs sample questions based on neet pg , usmle, pla...
Radiology most important signs sample questions based on neet pg , usmle, pla...
Medico Apps
 
23204980
2320498023204980
23204980
radgirl
 
Legg- Calve – Perthes disease
Legg- Calve – Perthes diseaseLegg- Calve – Perthes disease
Legg- Calve – Perthes disease
Phòng Khám An Nhi
 
Previous year question on bone cyst based on neet pg, usmle, plab and fmge or...
Previous year question on bone cyst based on neet pg, usmle, plab and fmge or...Previous year question on bone cyst based on neet pg, usmle, plab and fmge or...
Previous year question on bone cyst based on neet pg, usmle, plab and fmge or...
Medico Apps
 
Legg calve perthes disease
Legg calve perthes disease Legg calve perthes disease
Legg calve perthes disease
Ratan Khuman
 
Legg Calve Perthes disease
Legg Calve Perthes diseaseLegg Calve Perthes disease
Legg Calve Perthes disease
shahinhamza2
 
Nitin perthes
Nitin perthesNitin perthes

What's hot (20)

23205011
2320501123205011
23205011
 
23205056
2320505623205056
23205056
 
23205029
2320502923205029
23205029
 
23205062
2320506223205062
23205062
 
23205008
2320500823205008
23205008
 
23204964
2320496423204964
23204964
 
23204977
2320497723204977
23204977
 
23205065
2320506523205065
23205065
 
23204961
2320496123204961
23204961
 
23204955
2320495523204955
23204955
 
23204943
2320494323204943
23204943
 
23205001
2320500123205001
23205001
 
23204952
2320495223204952
23204952
 
Radiology most important signs sample questions based on neet pg , usmle, pla...
Radiology most important signs sample questions based on neet pg , usmle, pla...Radiology most important signs sample questions based on neet pg , usmle, pla...
Radiology most important signs sample questions based on neet pg , usmle, pla...
 
23204980
2320498023204980
23204980
 
Legg- Calve – Perthes disease
Legg- Calve – Perthes diseaseLegg- Calve – Perthes disease
Legg- Calve – Perthes disease
 
Previous year question on bone cyst based on neet pg, usmle, plab and fmge or...
Previous year question on bone cyst based on neet pg, usmle, plab and fmge or...Previous year question on bone cyst based on neet pg, usmle, plab and fmge or...
Previous year question on bone cyst based on neet pg, usmle, plab and fmge or...
 
Legg calve perthes disease
Legg calve perthes disease Legg calve perthes disease
Legg calve perthes disease
 
Legg Calve Perthes disease
Legg Calve Perthes diseaseLegg Calve Perthes disease
Legg Calve Perthes disease
 
Nitin perthes
Nitin perthesNitin perthes
Nitin perthes
 

Viewers also liked

First Partial Review
First Partial ReviewFirst Partial Review
First Partial Review
Carlos Vázquez
 
Koopia Uksusest Koolij Aniversay In Estonia Signe Leht
Koopia Uksusest Koolij  Aniversay In Estonia  Signe LehtKoopia Uksusest Koolij  Aniversay In Estonia  Signe Leht
Koopia Uksusest Koolij Aniversay In Estonia Signe Leht
LucianeCurator, EUNEOS Social Media Manager and Trainer
 
Lexgenda
LexgendaLexgenda
"5 Things in 5min" Series No.2 - Ms. CMO, 5 Reasons Why You Need Cloud
"5 Things in 5min" Series No.2 - Ms. CMO, 5 Reasons Why You Need Cloud"5 Things in 5min" Series No.2 - Ms. CMO, 5 Reasons Why You Need Cloud
"5 Things in 5min" Series No.2 - Ms. CMO, 5 Reasons Why You Need Cloud
Arun Cavale Cavale
 
Achieving Mastery Through Gamification
Achieving Mastery Through GamificationAchieving Mastery Through Gamification
Achieving Mastery Through Gamification
Pete Baikins
 
Small Business Project
Small Business ProjectSmall Business Project
Small Business Project
marshalls1
 
OEM International Group Presentation
OEM International Group PresentationOEM International Group Presentation
OEM International Group Presentation
OEMInternational
 
ממד
ממדממד
ממד
elio2b
 
Accounting charts
Accounting chartsAccounting charts
Accounting charts
David Jones
 
10 Thought Leaders Boiled Down 1204103143410416 4
10 Thought Leaders Boiled Down 1204103143410416 410 Thought Leaders Boiled Down 1204103143410416 4
10 Thought Leaders Boiled Down 1204103143410416 4kevin haynes
 
Environmental Group Of Support Sulkowice
Environmental Group Of Support  SulkowiceEnvironmental Group Of Support  Sulkowice
Environmental Group Of Support Sulkowice
LucianeCurator, EUNEOS Social Media Manager and Trainer
 
ap và_thuc_hanh_kinh_te_vi_moasdf
ap và_thuc_hanh_kinh_te_vi_moasdfap và_thuc_hanh_kinh_te_vi_moasdf
ap và_thuc_hanh_kinh_te_vi_moasdf
Đặng Trần Quyết
 
Basics of new twitter
Basics of new twitterBasics of new twitter
Basics of new twitter
Heidi Allen
 
DiffCalculus: September 18, 2012
DiffCalculus: September 18, 2012DiffCalculus: September 18, 2012
DiffCalculus: September 18, 2012
Carlos Vázquez
 
Teach tutorassescue2012
Teach tutorassescue2012Teach tutorassescue2012
Teach tutorassescue2012
National University
 
Virginia Empowerment Zone
Virginia Empowerment ZoneVirginia Empowerment Zone
Virginia Empowerment Zone
Catchphrasecopywriter
 
MOOCs - how to live with them and love them
MOOCs - how to live with them and love themMOOCs - how to live with them and love them
MOOCs - how to live with them and love them
Stephen Haggard
 
Ciclo Litúrgico
Ciclo LitúrgicoCiclo Litúrgico
Ciclo Litúrgico
cristinamoreubi
 
Logika Informatika
Logika InformatikaLogika Informatika
Logika Informatikaandrewaja
 

Viewers also liked (20)

First Partial Review
First Partial ReviewFirst Partial Review
First Partial Review
 
Koopia Uksusest Koolij Aniversay In Estonia Signe Leht
Koopia Uksusest Koolij  Aniversay In Estonia  Signe LehtKoopia Uksusest Koolij  Aniversay In Estonia  Signe Leht
Koopia Uksusest Koolij Aniversay In Estonia Signe Leht
 
Lexgenda
LexgendaLexgenda
Lexgenda
 
"5 Things in 5min" Series No.2 - Ms. CMO, 5 Reasons Why You Need Cloud
"5 Things in 5min" Series No.2 - Ms. CMO, 5 Reasons Why You Need Cloud"5 Things in 5min" Series No.2 - Ms. CMO, 5 Reasons Why You Need Cloud
"5 Things in 5min" Series No.2 - Ms. CMO, 5 Reasons Why You Need Cloud
 
Mkb gaat sociaal
Mkb gaat sociaalMkb gaat sociaal
Mkb gaat sociaal
 
Achieving Mastery Through Gamification
Achieving Mastery Through GamificationAchieving Mastery Through Gamification
Achieving Mastery Through Gamification
 
Small Business Project
Small Business ProjectSmall Business Project
Small Business Project
 
OEM International Group Presentation
OEM International Group PresentationOEM International Group Presentation
OEM International Group Presentation
 
ממד
ממדממד
ממד
 
Accounting charts
Accounting chartsAccounting charts
Accounting charts
 
10 Thought Leaders Boiled Down 1204103143410416 4
10 Thought Leaders Boiled Down 1204103143410416 410 Thought Leaders Boiled Down 1204103143410416 4
10 Thought Leaders Boiled Down 1204103143410416 4
 
Environmental Group Of Support Sulkowice
Environmental Group Of Support  SulkowiceEnvironmental Group Of Support  Sulkowice
Environmental Group Of Support Sulkowice
 
ap và_thuc_hanh_kinh_te_vi_moasdf
ap và_thuc_hanh_kinh_te_vi_moasdfap và_thuc_hanh_kinh_te_vi_moasdf
ap và_thuc_hanh_kinh_te_vi_moasdf
 
Basics of new twitter
Basics of new twitterBasics of new twitter
Basics of new twitter
 
DiffCalculus: September 18, 2012
DiffCalculus: September 18, 2012DiffCalculus: September 18, 2012
DiffCalculus: September 18, 2012
 
Teach tutorassescue2012
Teach tutorassescue2012Teach tutorassescue2012
Teach tutorassescue2012
 
Virginia Empowerment Zone
Virginia Empowerment ZoneVirginia Empowerment Zone
Virginia Empowerment Zone
 
MOOCs - how to live with them and love them
MOOCs - how to live with them and love themMOOCs - how to live with them and love them
MOOCs - how to live with them and love them
 
Ciclo Litúrgico
Ciclo LitúrgicoCiclo Litúrgico
Ciclo Litúrgico
 
Logika Informatika
Logika InformatikaLogika Informatika
Logika Informatika
 

Similar to 23205052

23204967
2320496723204967
23204967
radgirl
 
23204928
2320492823204928
23204928
radgirl
 
Impingement Femoroacetabular - Lee en forma critica...no todo lo que de dice ...
Impingement Femoroacetabular - Lee en forma critica...no todo lo que de dice ...Impingement Femoroacetabular - Lee en forma critica...no todo lo que de dice ...
Impingement Femoroacetabular - Lee en forma critica...no todo lo que de dice ...
Victor Olivares
 
Hip joint
Hip jointHip joint
Hip joint
Abdellah Nazeer
 
Scoliosis
ScoliosisScoliosis
Scoliosis
juhipatel1802
 
Transient Osteoporosis of Hip
Transient Osteoporosis of HipTransient Osteoporosis of Hip
Transient Osteoporosis of Hip
vinod naneria
 
Child Abuse_Section A3.pptx
Child Abuse_Section A3.pptxChild Abuse_Section A3.pptx
Child Abuse_Section A3.pptx
DeepakKumar313608
 
Slipped Capital Femoral Epiphysis
Slipped Capital Femoral EpiphysisSlipped Capital Femoral Epiphysis
Slipped Capital Femoral Epiphysis
Shady Mahmoud
 
Slide pfof
Slide pfofSlide pfof
Slide pfof
mustaqim wahit
 
Lateral condyle of humerus fracture in children
Lateral condyle of humerus fracture in childrenLateral condyle of humerus fracture in children
Lateral condyle of humerus fracture in children
AnilKC5
 
Muculoskeletal Pediatic Imaging..pptx
Muculoskeletal Pediatic Imaging..pptxMuculoskeletal Pediatic Imaging..pptx
Muculoskeletal Pediatic Imaging..pptx
Abdellah Nazeer
 
Orthopedic Aspects Of Metabolic Bone Disease By Xiu
Orthopedic Aspects Of Metabolic Bone Disease By XiuOrthopedic Aspects Of Metabolic Bone Disease By Xiu
Orthopedic Aspects Of Metabolic Bone Disease By Xiu
Xiu Srithammasit
 
Avascular necrosis
Avascular necrosisAvascular necrosis
Avascular necrosis
Ravish Ku Dewangan
 
16 hip disorders - d3
16   hip  disorders - d316   hip  disorders - d3
16 hip disorders - d3
Amr Mansour Hassan
 
subaxial cervical fx VI.pptx
subaxial cervical fx VI.pptxsubaxial cervical fx VI.pptx
subaxial cervical fx VI.pptx
williamvicky174
 
AAOS.2001.adult reconstruction.pdf
AAOS.2001.adult reconstruction.pdfAAOS.2001.adult reconstruction.pdf
AAOS.2001.adult reconstruction.pdf
alhassansaad1
 
Presentation1, radiological imaging of slipped femoral capital epiphysis.
Presentation1, radiological imaging of slipped femoral capital epiphysis.Presentation1, radiological imaging of slipped femoral capital epiphysis.
Presentation1, radiological imaging of slipped femoral capital epiphysis.
Abdellah Nazeer
 
Mandibular fracture
Mandibular fracture Mandibular fracture
Mandibular fracture
Abhishek PT
 
Spine injury -halim.pptx
Spine injury -halim.pptxSpine injury -halim.pptx
Spine injury -halim.pptx
ezrys54ety5
 
Nof fracture
Nof fractureNof fracture
Nof fracture
Dr Chinmoy Mazumder
 

Similar to 23205052 (20)

23204967
2320496723204967
23204967
 
23204928
2320492823204928
23204928
 
Impingement Femoroacetabular - Lee en forma critica...no todo lo que de dice ...
Impingement Femoroacetabular - Lee en forma critica...no todo lo que de dice ...Impingement Femoroacetabular - Lee en forma critica...no todo lo que de dice ...
Impingement Femoroacetabular - Lee en forma critica...no todo lo que de dice ...
 
Hip joint
Hip jointHip joint
Hip joint
 
Scoliosis
ScoliosisScoliosis
Scoliosis
 
Transient Osteoporosis of Hip
Transient Osteoporosis of HipTransient Osteoporosis of Hip
Transient Osteoporosis of Hip
 
Child Abuse_Section A3.pptx
Child Abuse_Section A3.pptxChild Abuse_Section A3.pptx
Child Abuse_Section A3.pptx
 
Slipped Capital Femoral Epiphysis
Slipped Capital Femoral EpiphysisSlipped Capital Femoral Epiphysis
Slipped Capital Femoral Epiphysis
 
Slide pfof
Slide pfofSlide pfof
Slide pfof
 
Lateral condyle of humerus fracture in children
Lateral condyle of humerus fracture in childrenLateral condyle of humerus fracture in children
Lateral condyle of humerus fracture in children
 
Muculoskeletal Pediatic Imaging..pptx
Muculoskeletal Pediatic Imaging..pptxMuculoskeletal Pediatic Imaging..pptx
Muculoskeletal Pediatic Imaging..pptx
 
Orthopedic Aspects Of Metabolic Bone Disease By Xiu
Orthopedic Aspects Of Metabolic Bone Disease By XiuOrthopedic Aspects Of Metabolic Bone Disease By Xiu
Orthopedic Aspects Of Metabolic Bone Disease By Xiu
 
Avascular necrosis
Avascular necrosisAvascular necrosis
Avascular necrosis
 
16 hip disorders - d3
16   hip  disorders - d316   hip  disorders - d3
16 hip disorders - d3
 
subaxial cervical fx VI.pptx
subaxial cervical fx VI.pptxsubaxial cervical fx VI.pptx
subaxial cervical fx VI.pptx
 
AAOS.2001.adult reconstruction.pdf
AAOS.2001.adult reconstruction.pdfAAOS.2001.adult reconstruction.pdf
AAOS.2001.adult reconstruction.pdf
 
Presentation1, radiological imaging of slipped femoral capital epiphysis.
Presentation1, radiological imaging of slipped femoral capital epiphysis.Presentation1, radiological imaging of slipped femoral capital epiphysis.
Presentation1, radiological imaging of slipped femoral capital epiphysis.
 
Mandibular fracture
Mandibular fracture Mandibular fracture
Mandibular fracture
 
Spine injury -halim.pptx
Spine injury -halim.pptxSpine injury -halim.pptx
Spine injury -halim.pptx
 
Nof fracture
Nof fractureNof fracture
Nof fracture
 

More from radgirl

23205042
2320504223205042
23205042
radgirl
 
23205036
2320503623205036
23205036
radgirl
 
23205025
2320502523205025
23205025
radgirl
 
23205019
2320501923205019
23205019
radgirl
 
23204998
2320499823204998
23204998
radgirl
 
23205004
2320500423205004
23205004
radgirl
 
23204995
2320499523204995
23204995
radgirl
 
23204989
2320498923204989
23204989
radgirl
 
23204992
2320499223204992
23204992
radgirl
 
23204986
2320498623204986
23204986
radgirl
 
23204983
2320498323204983
23204983
radgirl
 
23204970
2320497023204970
23204970
radgirl
 

More from radgirl (12)

23205042
2320504223205042
23205042
 
23205036
2320503623205036
23205036
 
23205025
2320502523205025
23205025
 
23205019
2320501923205019
23205019
 
23204998
2320499823204998
23204998
 
23205004
2320500423205004
23205004
 
23204995
2320499523204995
23204995
 
23204989
2320498923204989
23204989
 
23204992
2320499223204992
23204992
 
23204986
2320498623204986
23204986
 
23204983
2320498323204983
23204983
 
23204970
2320497023204970
23204970
 

23205052

  • 1. 2007 ACR Diagnostic Radiology In-Training Exam Rationales Section IX Musculoskeletal Radiology 179. You are shown a lateral radiograph of the spine and AP radiograph of the pelvis of a 60 year old man (Figures 1A and 1B). What is the MOST LIKELY diagnosis? A. Diffuse idiopathic skeletal hyperostosis B. Ankylosing spondylitis C. Psoriatic arthritis D. Ossification of the posterior longitudinal ligament RATIONALES: A. Correct. There is ossification of the anterior longitudinal ligament involving numerous (>4) levels with relative preservation of the disc spaces and there is extensive bone proliferation at the pelvis, at the trochanters, ischial tuberosities, iliac spines and iliac crests without sacroiliitis. These are typical features of DISH. The most common site of involvement is the vertebral column, Forestier’s disease, which characteristically involves the thoracic region. The preservation of the disc spaces and extensive spinal involvement help distinguish DISH from degenerative disc disease. Normal S-I joints exclude a diagnosis of AS which may otherwise demonstrate a similar proliferative enthesopathy at the pelvis. B. Incorrect. Sacroiliitis is a diagnostic criterion for ankylosing spondylitis and the S-I joints in this case are normal. The syndesmophytes of AS represent ossification of the annulus fibrosis of the disc and, therefore, are thinner and more vertical than the ossification of the anterior longitudinal ligament seen in DISH. C. Incorrect. The paravertebral ossification of Psoriatic arthritis, like that of Reiter’s syndrome, is relatively larger, bulky or irregular, unilateral or asymmetric in distribution and to the side of the vertebral column. The S-I joints may or may not be involved. Proliferative ensethopathy at the pelvis may be present. D. Incorrect. The posterior longitudinal ligament is not affected in this patient. Although ossification of the posterior longitudinal ligament (OPLL) may be seen at the thoracic spine, with or without cervical spine involvement, isolated cervical spine involvement is more typical. There are no associated abnormalities at the pelvis. References: Miller and Schweitzer. Diagnostic Musculoskeletal Imaging. McGraw-Hill, NY, 2005. Resnick. Diagnosis of Bone and Joint Disorders. 4th ed. W.B. Saunders, NY, 2002
  • 2. 2007 ACR Diagnostic Radiology In-Training Exam Rationales 180. You are shown an axial fat-suppressed fast spin echo T2-weighted image of a 16-year-old boy following knee trauma (Figure 2). What is the MOST LIKELY diagnosis? A. Ruptured anterior cruciate ligament B. Hyperextension injury C. Patellofemoral dislocation D. Direct impaction (“dashboard”) injury RATIONALES: A. Incorrect. Rupture of the ACL, caused by a valgus rotational injury, is associated with bone contusions at the lateral aspect of the knee, consisting of the weight-bearing portion of the lateral femoral condyle and the posterior aspect of the lateral tibial plateau. B. Incorrect. Hyperextension of the knee causes “kissing” bone bruises of the anterior aspect of the femoral condyles and tibial plateau. C. Correct. There is impaction fracture at the medial aspect of the patella, bone bruise at the lateral aspect of the lateral femoral condyle and partial tear of the medial patella retinaculum. As the patella dislocates laterally, the medial retinaculum is stretched. As the patella reduces spontaneously, the medial aspect of the patella impacts the lateral aspect of the lateral femoral condyle. This is often not suspected clinically. D. Incorrect. A direct blow to the anterior aspect of the knee may cause posterior dislocation of the femur or tibia. References: Sanders TG, Medynski MA, Feller JF, Lawhorn KW. Bone Contusion Patterns of the Knee at MR Imaging: Footprint of the Mechanism of Injury. Radiographics 2000;20:S135-S151
  • 3. 2007 ACR Diagnostic Radiology In-Training Exam Rationales 181. You are shown an AP radiograph of a 30-year-old woman (Figure 3). What structure is MOST LIKELY injured? A. Posterior cruciate ligament B. Iliotibial band C. Medial meniscus D. Anterior cruciate ligament RATIONALES: A. Incorrect. Posterior cruciate ligament tear is not a feature of this injury pattern. B. Incorrect. The iliotibial band inserts at Gerdy’s tubercle at the anterolateral aspect of the proximal tibia, distal and more anterior to this avulsed attachment of the lateral capsule. C. Incorrect. Both the medial and lateral menisci may be torn as part of the spectrum of injury, but not as often as the anterior cruciate ligament. D. Correct. The lateral bony fragment is a “Segond” fracture and is due to avulsion of the attachment of the lateral joint capsule resulting from a twisting injury associated with varus stress. It is almost always associated with rupture of the anterior cruciate ligament. References: Goldman AB, Pavlov H, Rubenstein D. The segond fracture of the proximal tibia: A small avulsion that reflects major ligamentous damage. AJR 1988; 151:1163- 1167
  • 4. 2007 ACR Diagnostic Radiology In-Training Exam Rationales 182. You are shown an oblique coronal T2-weighted gradient echo image of a 50-year-old man with shoulder pain (Figure 4). Which one of the following is the cause of the abnormality? A. Calcium hydroxyapatite B. Calcium pyrophosphate dihydrate C. Monosodium urate D. Hemosiderin RATIONALES: A. Correct. There is focal signal void at the suprapinatus tendon The capsular, tendinous, ligamentous and bursal tissues about the shoulder are the most common sites of peri-articular calcific deposits. The supraspinatus tendon is the most frequent site of calcification, usually at the insertion near the greater tuberosity. Although other crystals may deposit here, most are calcium hydroxyapatite. B. Incorrect. CPPD deposition may be present in synovium, cartilage, capsule, tendon, ligament and bursae. Involvement of the supraspinatous tendon is not uncommon and there may be associated HAA deposition. Deposition in cartilage, however, is more typical. C. Incorrect. Monosodium urate deposition may occur in the peri-articular soft tissues including the joint capsule, tendon, ligament and bursa but is much more common in the synovium and articular cartilage. There is a predilection for the lower extremity. D. Incorrect. Hemosiderin deposits within the synovium not the tendons and may be secondary to resolving hematoma or recurrent hemarthrosis as in pigmented villonodular synovitis or hemophilia. It is noted with inflammatory arthritis as well. References: Manaster, Disler, May. Musculoskeletal Imaging. The Requisites. 2nd ed. Mosby, St. Louis MO, 2002. Resnick, Niwayama. Diagnosis of Bone and Joint Disorders. W.B. Saunders. 2002,Philadelphia, PA. Fourth Ed.
  • 5. 2007 ACR Diagnostic Radiology In-Training Exam Rationales 183. You are shown an AP radiograph of the left hip of a 75-year-old man with hip pain (Figure 5). What is the MOST LIKELY diagnosis? A. Giant cell tumor B. Transient osteoporosis of the hip C. Plasmacytoma D. Paget’s Disease RATIONALES: A. Incorrect. Giant cell tumor is a sub-articular, focal, lytic lesion, often eccentric, and demonstrating expansile remodeling. Patients are usually 20-45 years of age. Extension of the neoplasm into the shaft without associated deformity of the bone would be unusual. B. Incorrect. Transient osteoporosis is a self-limited disorder, associated with transient bone marrow edema. It may be the sequela of subchondral insufficiency fracture. The radiographic appearance is that of focal osteoporosis at the femoral head with less extensive involvement of the femoral neck and acetabulum without involvement more distally and without the advancing edge seen in the test case. C. Incorrect. Plasmacytoma is a common lytic lesion of the skeleton in older patients. The lesions however are more focal with bone destruction appearing as a geographic lytic area sometimes with expansile remodeling, none of which is seen in the test case. Diffuse osteopenia is another manifestation. D. Correct. The lytic phase of Paget’s disease, typically begins at the end of a bone, or an apophysis, and advances towards the other end with a well-defined advancing edge, blade-like, V or wedge or flame shaped in appearance. The lytic nature of the process is secondary to bone resorption, not destruction or replacement, and therefore contour abnormalities are not prominent as in the test case where the only deformity is secondary to fracture. References: Smith,Murphey et al. Radiologic Spectrum of Paget Disease of Bone and Its Complications with Pathologic Correlation. AFIP Archives. Radiogaphics 2002.22:1191- 1216. Greenspan. Orthopaedic Radiology. Lipincott Williams Wilkins. Philadelphia, PA. Third Ed. Resnick, Niwayama. Diagnosis of Bone and Joint Diaorders. W.B. Saunders. Philadelphia, PA. Fourth Ed, 2002.
  • 6. 2007 ACR Diagnostic Radiology In-Training Exam Rationales 184. You are shown coronal (Figure 6A) and axial (Figure 6B) fat-suppressed T1- weighted MR arthrographic images of a 24-year-old man with right shoulder pain. Which of the following is the MOST LIKELY diagnosis? A. Hill-Sachs lesion B. Bankart lesion C. Perthes lesion D. SLAP lesion RATIONALES: A. Incorrect. The Hill-Sachs fracture deformity is a fracture at the posterior lateral aspect of the humeral head, superiorly, acquired following anterior shoulder dislocation and subsequent impaction at the anterior inferior glenoid. It is best seen on axial images at the level of the coracoid or higher. It should not be confused with the normal posterior humeral groove, seen more distally. B. Incorrect. A Bankart lesion is an anterior labral avulsion at the inferior glenohumeral ligament attachment with rupture of the anterior scapular periosteum and marked periosteal stripping. It is associated with anterior shoulder dislocation. Although contrast is seen beneath the detached labrum, it is confined by the intact overlying periosteum. C. Correct. A Perthes lesion is a Bankart variant, an anterior labral avulsion at the inferior glenohumeral ligament attachment with intact scapular periosteum and mild periosteal stripping. The intra-articular contrast is seen beneath the labrum at the detachment site and extends medially beneath the mildly stripped, but otherwise intact, periosteum. D. Incorrect. The superior labrum is intact. References: Stoller DW. Shoulder. In: Stoler DW, Tirman PF, Bredella MA, eds. Orthopaedics. Salt Lake City, UT: Amirsys, 2004: 58-93.
  • 7. 2007 ACR Diagnostic Radiology In-Training Exam Rationales 185. A standard spin echo pulse sequence with TR = 4000 ms and TE = 90 ms will have image contrast chiefly dominated by ____________ weighting. A. T1 B. Proton density C. T2 X D. T2* RATIONALES: A. Incorrect: Standard spin-echo sequences rely on the fact that tissues with short (long) T1 will typically have a short (long) T2. T1 is the spin-lattice relaxation constant, which describes the time required for re-establishment of 63% of the longitudinal magnetization, and T2 is the spin-spin relaxation constant, which describes the time required for decay of the transverse magnetization to 37% of its original peak amplitude. T1 contrast is manifested by selecting a TR time that maximizes differences in the T1 characteristics of the tissues, and is typically between about 300-700 ms for a standard spin-echo sequence. In order to reduce the effects of T2 decay, a short TE (<10 ms) is required. The stem indicates values much longer than would generate T1 contrast. B. Incorrect: TR is considered “long” in a standard spin-echo pulse sequence above about 800 to 1000 ms, where the longitudinal magnetization differences are manifested chiefly as spin-density (proton-density) variations, with minimal T1 weighting. While a TR of 4000 ms can certainly result in spin-density weighting, the other part of signal generation is the spin-spin decay of transverse magnetization, which requires a very short TE (<10 ms). The TE of 90 ms is considered to be long, giving rise to differences in the T2 characteristics of the tissues. C. Correct: TR is considered “long” in a standard spin-echo pulse sequence greater than 800 to 1000 ms, where the longitudinal magnetization differences are manifested chiefly as spin-density (proton-density) variations, to reduce any T1 weighting effects. For TR = 4000 ms, there is little or no T1 weighting. Transverse magnetization losses (spin-spin decay) are due to T2 effects; by allowing the decay to occur over a relatively long time prior to producing an echo, more T2 contrast will result. For spin-echo sequences, TE > 50 ms is considered long, and will permit more transverse decay to occur, resulting in the manifestation of T2 contrast. D. Incorrect: T2* weighting is not apparent with a standard spin-echo pulse sequence because of the 180° refocusing pulse, which causes the de-phasing spins to be subject to external magnetic inhomogeneities in the opposite direction, which cancels the de- phasing effect in the reformed echoes.
  • 8. 2007 ACR Diagnostic Radiology In-Training Exam Rationales References: Bushberg JT, Seibert JA, Leidholdt EM, Boone JM. The Essential Physics of Medical Imaging, 2nd Edition, Chapter 14, p. 399. 186. Concerning sacroiliitis, which of the following is MOST LIKELY to be mono- articular? A. Tuberculosis B. Progressive systemic sclerosis C. Inflammatory bowel disease D. Reiter’s syndrome RATIONALES: A. Correct. Tuberculous spondylitis is the most common form of skeletal tuberculosis. Extra-spinal articular disease typically involves large joints such as the knee and hip but any joint may be involved. Monoarticular involvement is characteristic. Osteomyelitis without septic arthritis is relatively uncommon. B. Incorrect. Sacroiliitis is not a feature of progressive systemic sclerosis or scleroderma. C. Incorrect. The sacroiliitis associated with ulcerative colitis and Crohn’s disease is identical to that of ankylosing spondylitis, bilateral and symmetric. D. Incorrect. Sacro-iliitis in Reiter’s syndrome is common. The involvement tends to be bilateral, symmetric or asymmetric. Asymmetric and rarely,unilateral involvement is more common in Reiter’s syndrome and psoriatic arthritis than ankylosing spondylitis. References: Miller and Schweitzer. Diagnostic Musculoskeletal Imaging. McGraw-Hill, NY, 2005. Chew. Skeletal Radiology: The Bare Bones. 2nd ed. Williams and Wilkins, Baltimore, 1997. Resnick. Diagnosis of Bone and Joint Disorders.4th ed. W.B. Saunders, NY, 2002
  • 9. 2007 ACR Diagnostic Radiology In-Training Exam Rationales 187. Concerning chronic renal disease, which of the following is MOST closely associated? A. Neuropathic osteoarthropathy B. Tarsal tunnel syndrome C. Destructive spondyloarthropathy D. Hypertrophic osteoarthropathy RATIONALES: A. Incorrect. Neuropathic osteoarthropathy may result from several underlying disorders including diabetes mellitus, syphilis, and numerous congenital insensitivity to pain syndromes. Radiographic findings of bone destruction and fragmentation may mimic the destructive spondyloarthropathy associated with renal dialysis. B. Incorrect. Carpal tunnel syndrome is a known complication of patients undergoing hemodialysis. This has been attributed to edema, venous congestion and subsequent compression of the median nerve related to the access site. Amyloid deposition, B2-microglobulin, in the synovium also results in such compression. Tarsal tunnel syndrome is not a clinical manifestation related to chronic renal disease. C. Correct. Patients undergoing renal dialysis for at least 2-3 years may develop a destructive spondyloarthropathy, most common at the cervical region. This may or may not be symptomatic. Radiographic findings of disc space loss, endplate erosion and reactive sclerosis mimic septic spondylitis, neuropathic spondyloarthropathy and CPPD deposition disease. Although chronic hyperparathyroidism and ligamentous laxity may be contributing factors, amyloid deposition seems primary. D. Incorrect. Hypertrophic osteoarthropathy, usually associated with pulmonary disorders (hypertrophic pulmonary osteoarthropathy, HPO) most notably bronchogenic carcinoma, is a triad of painful swollen joints, clubbing and periosteal new bone formation. Although this is associated with numerous disorders including inflammatory bowel disease, biliary cirrhosis, chronic lung diseases, and congenital cyanotic heart disease, there is no association with chronic renal disease. References: Miller and Schweitzer. Diagnostic Musculoskeletal Imaging. McGraw-Hill, NY, 2005. Manaster, Disler, May. Musculoskeletal Imaging. The Requisites. 2nd ed. Mosby, St. Louis, 2002. Resnick. Diagnosis of Bone and Joint Disorders.4th ed. W.B. Saunders, NY, 2002
  • 10. 2007 ACR Diagnostic Radiology In-Training Exam Rationales 188. Concerning mixed connective tissue disease, which of the following is TRUE? A. Males and females are equally affected. B. It is a combination of scleroderma, systemic lupus erythematosus, and polyarteritis nodosa. C. Erosive disease is not characteristic. D. Serology is essential to diagnosis. RATIONALES: A. Incorrect. Approximately 80% of patients are woman. B. Incorrect. Mixed connective tissue disease (MCTD) is a disorder characterized by clinical abnormalities typical of SLE (systemic lupus erythematosus), PSS (progressive systemic sclerosis or scleroderma), dermatomyositis and rheumatoid arthritis. C. Incorrect. Joint involvement is typical resembling the changes of rheumatoid arthritis. Occasionally, nonerosive deformities similar to SLE are seen. D. Correct. The one feature of MCTD which distinguishes it as a unique disorder is a positive serologic test for antibody to the ribonucleoprotein (RNP) of extractable nuclear antigen (ENA). References: Miller and Schweitzer. Diagnostic Musculoskeletal Imaging. McGraw-Hill, NY, 2005. Greenspan, A. Orthopedic Radiology: A Practical Approach. Lippincott Williams Wilkins, 3rd . ed., 2000.
  • 11. 2007 ACR Diagnostic Radiology In-Training Exam Rationales 189. Concerning Blount’s disease, which of the following is TRUE? A. It is also known as tibia valga. B. Radiographic abnormalities are evident in the first 2 years of life. C. The disorder can occur in infants, children and adolescents. D. It is usually self-limited and requires no treatment. RATIONALES: A. Incorrect. Blount’s disease is also known as tibia vara. The disorder affects the medial aspect of the proximal tibia. The deformity consists of varus angulation and internal rotation at the proximal tibial metaphysis. Factors contributing to the disorder include varus stress, focal growth suppression and disruption of endochondral ossification. B. Incorrect. Radiographic abnormalities in the infantile form rarely are evident before 2 years of age at which time they resemble physiologic bowing. The varus deformity, however, occurs at the metaphysis, not the knee. Eventually, the medial metaphysis becomes depressed and an osseous excrescence or outgrowth may develop. C. Correct. The early onset or infantile group occurs in children less than three years of age. The late onset group consists of a juvenile and an adolescent form. Deformity is more likely in the younger age groups. D. Incorrect. The natural history of the disease is that of irreversible deformity resulting from changes at the growth plate. In the infantile form, orthotic bracing is usually the first line of treatment. If this fails, an osteotomy is required. In the adolescent form, treatment is usually surgical. References: Bradway JK, Klassen RA, Peterson HA. Blount disease: a review of the English literature. J Pediatr Orthop 1987; 7(4):472-480 Blounts Disease. Wheeless' Textbook of Orthopaedics. Duke Orthopaedics. www.wheelessonline.com/ortho/blounts_disease Resnick, Niwayama. Diagnosis of Bone and Joint Disorders. W.B. Saunders. 2002,Philadelphia, PA. Fourth Ed
  • 12. 2007 ACR Diagnostic Radiology In-Training Exam Rationales 190. Concerning aneurysmal bone cysts, which of the following are associated? A. Fibrous dysplasia B. Osteoid osteoma C. Enchondroma D. Plasmacytoma RATIONALES: A. Correct. Aneurysmal bone cysts are often superimposed upon existing benign and malignant tumors most notably giant cell tumor, nonossifying fibroma, fibrous dysplasia, chondroblastoma, osteoblastoma, osteosarcoma, chondrosarcoma and hemangioendothelioma. In the case of telangiectatic osteosarcoma, the two different components of the lesion may appear similar or identical with imaging. This may result in errors with percutaneous biopsy if only portions of the cyst are sampled. Nonossifying fibroma is a benign, cortically based lesion that is usually found in long tubular bones. The etiology is argued, but many feel the nonossifying fibroma to be the sequelae of unrecognized trauma to the periosteum that leads to focal hemorrhage. Aneurysmal bone cysts are also felt by many to represent prior trauma, not neoplasia. It is therefore intuitive that the two lesions could be found in the same location. Fibrous dysplasia is a hamartomatous disorder of osteoblasts that fail to undergo differentiation. The monostotic form is most common. Skin and endocrine anomalies may be associated with the polyostotic form. Although fibrous dysplasia may rarely be associated with malignant transformation, new symptoms or growth of a known focus of fibrous dysplasia often signifies the development of a superimposed aneurysmal bone cyst. B. Incorrect. Osteoid osteoma is a benign bone forming neoplasm usually cortically based and extra-articular, with characteristic reactive sclerosis. Intracapsular, intramedullary and subperiosteal lesions also occur. Osteoid osteomas that are intra-articular incite little or no sclerosis. Intra-medullary and subperiosteal lesions provoke variable degrees of sclerosis. There is no association with ABC. Osteoblastoma, however, is associated. C. Incorrect. Enchondroma is a benign cartilage forming neoplasm within the medullary canal. Such lesions on the surface of the bone are known as superiosteal or juxta-cortical chondromas. They are most common at the fingers. There is no association with ABC. D. Incorrect. Plasmactyoma represents the solitary form of myeloma. While laboratory teats may be negative at the time of diagnosis, many patients will eventually develop multiple myeloma. The lesions are usually lytic, and may be expansile or non- expansile. Rarely, a plasmacytoma may be blastic. An association between plasmacytoma and aneurysmal bone cyst has not been described.
  • 13. 2007 ACR Diagnostic Radiology In-Training Exam Rationales References: Mirra JM, Picci P, Gold RH. Bone Tumors: Clinical, Radiologic, and Pathologic Correlations. Lea & Febiger, Philadelphia, PA, 1989 191. Which of the following tumors is characterized by a subarticular location? A. Osteoblastoma B. Clear cell chondrosarcoma C. Chondromyxoid fibroma D. Aneurysmal bone cyst RATIONALES: A. Incorrect. The most common location for osteoblastoma is in the dorsal elements of the spine where forty percent are found. The remaining are usually located in the long bones and phalanges. An epiphyseal or subarticular location is rare. Osteoblastoma is usually expansile, and may be associated with neurologic deficit. B. Correct. Few lesions have a propensity for the end of the bone, epiphysis or apophysis (epiphysis equivalent). Giant cell tumor of bone and chondroblastoma are oth such lesions. Approximately 5% of giant cell tumors are malignant and associated with lung metastases. Otherwise, both of these are considered benign. The malignant tumor that is found at the end of the bone, subarticular, is clear cell chondrosarcoma. This is a slow growing tumor, and symptoms may be present for over five years before diagnosis. The tumor may be lobulated and calcified, or lytic and ill defined. C. Incorrect. As the name implies, chondromyxoid fibroma is a benign tumor composed of chondroid, myxoid and fibrous tissue. Three-fourths are found in the lower extremities. Approximately half involve long bones, and ninety-five percent of those are found in the metaphysis. The remaining five percent are diaphyseal. D. Incorrect. Although aneurysmal bone cysts may be engrafted on epiphyseal or subarticular lesions such as giant cell tumor and chondroblastoma, the most common locations are the metaphysis of long bones and the dorsal elements of the vertebrae. Approximately one third of aneurysmal bone cysts are secondary, associated with other entities. The remaining may represent primary aneurysmal bone cysts, or may reflect complete destruction of the entire pre-existing tumor by the aneurysmal bone cyst. References:
  • 14. 2007 ACR Diagnostic Radiology In-Training Exam Rationales Mirra JM, Picci P, Gold RH. Bone Tumors: Clinical, Radiologic, and Pathologic Correlations. Lea & Febiger, Philadelphia, PA, 1989 192. Concerning osteosarcoma, what type has the worst prognosis? A. Secondary B. Telangiectatic C. Periosteal D. Parosteal RATIONALES: A. Correct. Secondary osteosarcoma may be seen following radiation therapy, or may be associated with underlying bone lesions such as Paget's disease, fibrous dysplasia, bone infarction and chronic osteomyelitis. Other types of sarcoma may also be seen with these conditions, including fibrosarcoma and malignant fibrous histiocytoma. The prognosis for these secondary sarcomas is often grave. B. Incorrect. Telangiectatic osteosarcoma is an entirely lytic lesion, reflecting minimal osteoid production in the tumor. Far less common than conventional osteosarcoma, it was previously felt that the telangiectatic form carried a worse prognosis. More recent studies have shown a similar or even better prognosis, as the telangiectatic form is especially sensitive to preoperative chemotherapy. C. Incorrect. Periosteal osteosarcoma is a chondroblastic form of osteosarcoma that arises either in the periosteum or in the outer cortex, and is usually diaphyseal in location. The lesions show a sunburst pattern of mineralization. They are usually considered to be intermediate grade, between high grade central and low grade parosteal osteosarcomas. D. Incorrect. The parosteal (juxtacortical) osteosarcoma is found on the outer surface of the cortex. These tumors are well differentiated, with a prognosis better than other forms of osteosarcoma. A long-tern survival of eighty to ninety percent is seen if the tumor has not de-differentiated. Two thirds of parosteal osteosarcomas are located along the distal femoral shaft. Most are heavily ossified. References: Unni KK, ed. Dahlin's Bone Tumors: General Aspects and Data on 11, 087 cases, 5th ed. Lippincott-Raven Publishers, Philadelphia, PA, 1996
  • 15. 2007 ACR Diagnostic Radiology In-Training Exam Rationales 193. Concerning osteopoikilosis, which of the following is TRUE? A. Patients often complain of restricted range of motion at the joints involved. B. Histologically, the lesions are easily confused with metastatic prostate cancer. C. The patient’s serum alkaline phosphatase may be mildly elevated. D. Transmission is autosomal dominant. RATIONALES: A. Incorrect. Osteopoikilosis is characterized by multiple bone islands or enostoses with a periarticular distribution. This is an asymptomatic, incidental condition not to be confused with other sclerotic lesions most notably blastic metastases. It may be considered a type of sclerosing dysplasia similar in this regard to osteopathia striata (Voorhoeve’s disease). B. Incorrect. Histologically, the lesions are benign bone islands. They consist of compact lamellar bone with haversian systems. The bone is uniform with regular cement lines and no evidence of cartilage. The characteristic radiographic appearance includes numerous, small, round or ovoid (long axis parallel to long tubular bone) sclerotic lesions, with spiculated margins and periarticular, symmetric distribution favoring long tubular bones, the carpus, tarsus, pelvis and scapulae. Lesions may increase or decrease in size and number although this phenomenon is more common in children. Bone scans are usually normal. C. Incorrect. Laboratory examinations are normal. D. Correct. Both inherited and sporadic cases of osteopoikilosis have been described. There is an autosomal dominant pattern of transmission and penetrance is high. References: Lagier R, Mbakop A, Bigler A. Osteopoikilosis: a radiological and pathological study. Skeletal Radiol 1984; 11: 161-168 Resnick, Niwayama. Diagnosis of Bone and Joint Disorders. W.B. Saunders. 2002, Philadelphia, PA. Fourth Ed
  • 16. 2007 ACR Diagnostic Radiology In-Training Exam Rationales 194. Which of the following synovial spaces normally communicate with each other? A. The glenohumeral joint and the subacromial bursa B. The pisiform recess and the radiocarpal joint C. The ankle joint and the peroneal tendon sheath D. The posterior subtalar joint and the Achilles tendon sheath RATIONALES: A. Incorrect. Communication between the glenohumeral joint and the subacromial bursa (more accurately, the subacromial/subdeltoid bursal complex) indicates the presence of a full thickness rotator cuff tear. This principal is the basis for diagnosis of such tears on both conventional and MR glenohumeral arthrography. B. Correct. Either space may be entered for wrist arthrography. Excessive fluid in the pisiform recess should not be considered abnormal if there is a radiocarpal joint effusion. C. Incorrect. The ankle joint may normally communicate with the tendon sheaths of the flexor digitorum longus and flexor hallucis longus at the medial aspect of the ankle. Communication with the lateral tendon sheath (common peroneal) implies a tear of the calcaneofibular ligament. D. Incorrect. The Achilles tendon does not have a true tendon sheath. Instead, it is surrounded by connective tissue called a paratenon. This provides for the normal gliding action of the tendon. References: Arndt RD, Horns JW, Gold RH. Clinical Arthrography. 2nd ed. Williams & Wilkins, Baltimore, MD. 1985.
  • 17. 2007 ACR Diagnostic Radiology In-Training Exam Rationales 195. Concerning pars interarticularis defects, which of the following is TRUE? A. Symptomatic spondylolisthesis is typically associated. B. Associated spondylolisthesis is most common at L5-S1. C. They represent an acute injury. D. Magnetic resonance imaging is the preferred imaging method. RATIONALES: A. Incorrect. Only approximately twenty-five percent of patients with spondylolysis develop spondylolisthesis. Progression to significant slippage is uncommon. B. Correct. Spondylolisthesis associated with spondylolysis is most common at L5-S1, approximately 90% of cases. The remainder usually occur at L4-L5. Spondylolisthesis associated with degenerative disease is most common at L4- L5. C. Incorrect. Pars interarticularis defects are considered to represent fatigue stress fractures. Most are believed to develop in childhood. They are especially common in athletic children. D. Incorrect. Imaging should begin with conventional radiographs. The lateral and oblique views are nearly equally sensitive in diagnosing the defect. CT also easily displays the lesion. SPECT imaging can be useful in identifying pars interarticularis defects that are symptomatic. Magnetic resonance imaging plays little role. Difficulty in recognizing an intact pars interarticularis leads to a low positive predictive value. References: Garry JP, McShane J. Lumbar spondylolysis in adolescent athletes. J Fam Pract. 1998; 47(2): 145-149 Standaert CJ, Herring SA. Spondylolysis: a critical review. Br J Sports Med 2000; 34(6): 415-422 Resnick. Diagnosis of Bone and Joint Disorders.4th ed. W.B. Saunders, NY, 2002
  • 18. 2007 ACR Diagnostic Radiology In-Training Exam Rationales 196. Concerning the vertebral column, which of the following is TRUE? A. Paget’s Disease spares the posterior elements. B. Hematogenous infection preferentially seeds the vertebral body. C. Soft tissue extension excludes the diagnosis of hemangioma. D. The inflammatory spondyloarthropathies spare the intervertebral discs. RATIONALES: A. Incorrect. Paget‘s disease of the spine typically involves the vertebral body and the posterior elements. This may be a helpful diagnostic clue when differentiating Paget ’s disease, blastic metastases/ lymphoma and hemangioma. B. Correct. Hematogenous infection of the spine begins as an osteomyelitis near one of the vertebral body endplates. Typically, the infection then spreads across the disc to the adjacent vertebral body resulting in destruction and erosion of the endplates and disc. Less commonly, the infection may spare the disc tracking beneath the anterior or posterior longitudinal ligaments, a pattern usually seen with tuberculous involvement. Despite the use of the term “discitis,” infection involving the spinal column rarely originates within the disc itself. C. Incorrect. Large hemangiomata may extend beyond the cortex into the para vertebral soft tissues and be symptomatic. D. Incorrect. The inflammatory spondyloarthropathies comprise the sero-negative disorders which typically involve synovial, cartilaginous and fibrous joints. The cartilaginous joints in the adult skeleton are the intervertebral discs and the symphysis pubis. Syndesmophyte formation and subsequent ossification of the intervertebral disc in patients with ankylosing spondylitis is an example of such involvement. References: Resnick,Niwayama. Diagnosis of Bone and Joint Disorders. W B Saunders,Philadelphia, PA . Fourth Ed. 2002
  • 19. 2007 ACR Diagnostic Radiology In-Training Exam Rationales 197. Concerning the osteochondroses, which of the following entities represents normal development? A. Kohler’s B. Scheurermann’s C. Kienbock’s D. Sever’s RATIONALES: A. Incorrect. The osteochondroses compromise a group of entities characterized by involvement of the epiphysis, apophysis or epiphyseoid bone with radiographic findings of fragmentation, collapse and sclerosis suggesting osteonecrosis. Although some of these entities represent osteonecrosis, others are the sequela of abnormal stress and others represent normal development. Kohler’s disease involves the tarsal navicular and is rare. It is a self-limited disorder, difficult to distinguish from variations of normal ossification. When a child is symptomatic and radiographic findings of flattening and sclerosis are detected in a previously normal navicular bone, the diagnosis of osteonecrosis is more certain. B. Incorrect. Scheuermann’s disease represents a growth disturbance of the spine characterized by multilevel anterior vertebral body wedging, vertebral body endplate irregularities with Schmorl’s node formation and increasing thoracic kyphosis. These changes are variable and most likely secondary to stress related intraosseous displacement of disc material (cartilagenous node formation) through cartilaginous endplates weakened on a congenital or traumatic basis. C. Incorrect. Kienbock’s disease represents osteonecrosis of the lunate seen in patients 20-40 years of age. D. Correct. Sever’s phenomenon represents normal ossification of the calcaneal apophysis. References: Resnick,Niwayama. Diagnosis of Bone and Joint Disorders. W B Saunders,Philadelphia, PA . Fourth Ed. 2002
  • 20. 2007 ACR Diagnostic Radiology In-Training Exam Rationales 198. Concerning cervical spine fracture, which is characteristically associated with acute, severe neurologic injury? A. Jefferson B. Extension teardrop C. Hangman’s D. Flexion teardrop RATIONALES: A. Incorrect. In a classic Jefferson fracture, the transverse ligament is intact and no instability is present and because displacement of the bony fragments occurs in a centripetal pattern during axial loading, cord damage is uncommon. In one large series, no patient presented with neurologic symptoms. B. Incorrect. This fracture is an avulsion fracture that arises from the anteroinferior corner of the vertebral body caused by a hyperextension injury. Retrolisthesis is often present but of minimal degree and only about 9% of patients present with neurologic symptoms. C. Incorrect. Bilateral fracture of the C2 pars interarticularis typically results from hyperextension. Death from judicial hanging resulted from delayed extension/distraction. Most cases today are secondary to motor vehicle accidents with transient hyperextension and no distraction. There is usually anterior subluxation of C2 on C3. Although unstable by nature, neurologic deficits are uncommon. Without significant distraction, the cord is typically spared because the acquired pars fracture allows for canal widening at a level where the cord already has abundant room. D. Correct. The flexion teardrop fracture is the most severe flexion injury characterized by complete disruption of all ligaments, intervertebral disc and facet joints at the level of injury and a large triangular fracture fragment consisting of the anterior, inferior aspect of the involved vertebral body. There is neither ligamentous or skeletal stability. This completely unstable fracture is associated with severe neurologic symptoms in 87% of patients including complete quadriplegia, paraplegia, Brown- Sequard syndrome, or anterior cord syndrome. It is caused by combined flexion and axial loading and classically affects C5. References: Resnick, D. Physical Injury: Spine. In: Resnick D, ed. Diagnosis of Bone and Joint Disorders. 4th ed. Philadelphia, PA: W.B. Saunders, 2002: 2958-2978. Yu JS. Hyperflexion teardrop fracture. In: Case Review – Musculoskeletal Imaging. St. Louis, MO: Mosby, 2001:127-128.
  • 21. 2007 ACR Diagnostic Radiology In-Training Exam Rationales 199. Concerning injuries of the wrist, which is MOST severe? A. Lunate dislocation B. Scapholunate dissociation C. Dorsal intercalated segment instability D. Perilunate dislocation RATIONALES: A. Correct. The dislocation of carpal bones about the lunate, with or without fracture, usually results from dramatic wrist hyperextension. High velocity trauma or falls from substantial heights may produce forceful palmar tension and dorsal compression that exceeds the limits of ligament and/or bony carpal constraints. The progressive perilunar instability model for such injury predicts sequential disruption of ligamentous attachments, forces transmitted from the radial to the ulna aspect of the carpus. In stage I, there is disruption of the scapho-lunate ligament and subsequent scapho-lunate dissociation. In stage II, the capitate and scaphoid separate from the lunate and triquetrum . In stage III, there is lunotriquetral dissociation allowing the entire carpus to separate from the lunate almost always with dorsal displacement, hence dorsal perilunate dislocation. In stage IV, the dorsal extrinsic ligaments fail and the lunate may dislocate volarly. This represents the most severe form and highest degree of instability. Such transmission of force purely about the lunate follows the so-called lesser arc pathway. Perilunar forces may also be transmitted through adjacent bone, so- called greater arc injuries. The scaphoid, trapezium, capitate, hamate and triquetrum may fracture. As one might expect, there may be concomitant fracture and ligamentous injury resulting in the perilunar dislocation stages described. The lesser and greater arcs together comprise the vulnerable zone for all such injuries. B. Incorrect. Scapholunate dissociation represents a stage I lesser arc injury and therefore is not considered severe. It is, however, a component of scapholunate instability, the most frequent form of carpal instability. In its most general sense, instability refers to a clinical perception by the patient that the joint can not withstand normal loads. Scapholunate dissociation refers to the anatomic, ligamentous disruption between scaphoid and lunate. This may only be detectable with stress radiographs or fluoroscopy and hence is referred to as dynamic instability. This may be readily apparent with routine radiographs, hence static instability. The scapholunate angle may be normal as seen on lateral radiographs. Only when the scaphoid assumes a flexed position however, is there rotatory subluxation. C. Incorrect. An intercalated segment is a middle segment of a three segment system under compression. This middle segment is inherently unstable unless stabilized by its connection to the other two segments, hence the proximal carpal row. DISI (dorsal intercalated segment instability) refers to the dorsiflexed posture or extension of all or part of the proximal carpal row, relative to the radius
  • 22. 2007 ACR Diagnostic Radiology In-Training Exam Rationales and capitate. This may be associated with numerous injuries, with and without dissociation of intrinsic or interosseous ligaments. It is less severe than lunate dislocation. D. Incorrect. Peri-lunate dislocation is a stage III perilunar injury and, therefore, less severe than stage IV lunate dislocation. References: Resnick, D. Physical Injury: Extraspinal Sites. In: Resnick D, ed. Diagnosis of Bone and Joint Disorders. 4th ed. Philadelphia, PA: W.B. Saunders, 2002: 2837-2842. Viegas SF, Patterson RM, Ward K. Extrinsic wrist ligaments in the pathomechanics of ulnar translation instability.J Hand Surg. 1995; 20A: 312-318. Yu JS. Trans-scaphoid perilunate dislocation. In: Case Review – Musculoskeletal Imaging. St. Louis, MO: Mosby, 2001:129-130. Cooney,WP et at. The Wrist: Diagnosis and Operative Treatment. St. Louis, MO: Mosby, 1998. 200. Portable radiographs taken with a film-screen system using a fixed radiographic grid tend to have less contrast than the radiographs taken in the radiography rooms. Which of the following factors is the MOST LIKELY cause for the reduced contrast? A. Use of lower kVp B. Use of higher mAs C. Use of Lower grid ratio D. Use of higher speed film-screen system RATIONALES: A. Incorrect. Lower kVp would increase contrast. B. Incorrect. Higher mAs would darken film, but not change contrast. C. Correct. Lower grid ratio radiographic grid is used to minimize cutoff from poor alignment, however the lower grid ratio yields less cleanup of the scatter radiation. D. Incorrect. Higher speed film-screen does not necessarily reduce contrast. References: A. Wolbarst, Physics of Radiology (1993), Chapter 20 J.T. Bushberg, et al., The Essential Physics of Medical Imaging (2002), Chapter 6
  • 23. 2007 ACR Diagnostic Radiology In-Training Exam Rationales 201. Which of the following structures is essential to maintaining the arch of the foot? A. Plantar fascia B. Posterior tibial tendon C. Spring ligament D. Peroneal brevis tendon RATIONALES: A. Incorrect. The plantar fascia assists in supporting the midfoot arch but is considered a secondary support. Rupture does not produce significant separation of the plantar fascia and does not result in flatfoot deformity. The clinical presentation is one of pain and swelling at the heel. B. Correct. Complete rupture of the posterior tibial tendon frequently results in pes planus deformity, the posterior tibial tendon being a primary support of the midfoot arch. It is opposed by the peroneus brevis tendon which everts the heel and abducts the foot. Posterior tibial tendon rupture allows the unopposed peroneal brevis tendon to result in hindfoot valgus and forefoot abduction. Once the calcaneus is in a valgus position, the Achilles tendon will further evert the calcaneus. The talo-navicular joint is gradually disrupted as all the medial ligaments become stretched and elongated. C. Incorrect. The spring ligament extends from the sustentaculum tali to the plantar aspect of the navicular. It supports the talar head and therefore the longitudinal arch of the foot. It is thick and strong and almost never ruptures during routine foot/ankle trauma. Although the spring ligament serves as a secondary support of the midfoot arch, it is not a primary support and as long as the posterior tibialis tendon is intact, the midfoot does not collapse. D. Incorrect. As discussed, rupture of the peroneal brevis tendon would allow the posterior tibial tendon to function unopposed . This injury is caused by repeated ankle sprain or chronic peroneal subluxation. It does not lead to a pes planus deformity. References: Yu JS. Posterior tibial tendon tear. In: Case Review – Musculoskeletal Imaging. St. Louis, MO: Mosby, 2001:107-108. Schweitzer ME, Karasick D. MR imaging of disorders of the posterior tibialis tendon. AJR Am J Roentgenol. 2000; 175: 627-35. Yu JS. Pathologic and post-operative conditions of the plantar fascia: review of MR imaging appearances. Skeletal Radiol. 2000; 29: 491-501.